Download as pdf or txt
Download as pdf or txt
You are on page 1of 76

1/3 Question 1-3 of 87

Theme: Spinal disorders

A. Osteomyelitis
B. Potts disease of the spine
C. Scheuermanns disease
D. Transverse myelitis
E. Tabes dorsalis
F. Subacute degeneration of the cord
G. Brown-Sequard syndrome
H. Syringomyelia
I. Epidural haematoma

Which is the most likely diagnosis for the scenario given. Each option may be used once, more than once or
not at all.

1. A 68 year old man presents to the plastics team with severe burns to his hands. He is not distressed
by the burns. He has bilateral charcot joints. On examination there is loss of pain and temperature
sensation of the upper limbs.

You answered Transverse myelitis

The correct answer is Syringomyelia

This patient has syringomyelia which selectively affects the spinotholamic tracts.Syringomyelia is a
disorder in which a cystic cavity forms within the spinal cord. The commonest variant is the Arnold-
Chiari malformation in which the cavity connects with a congenital malformation affecting the
cerebellum. Acquired forms of the condition may occur as a result of previous meningitis, surgery or
tumours. Many neurological manifestations have been reported, although the classical variety spares
the dorsal columns and medial lemniscus and affecting only the spinothalamic tract with loss of pain
and temperature sensation. The bilateral distribution of this patients symptoms would therefore favor
syringomyelia over SCID or Brown Sequard syndrome. Osteomyelitis would tend to present with
back pain and fever in addition to any neurological signs. Epidural haematoma large enough to
produce neurological impairment will usually have motor symptoms in addition to any selective
sensory loss, and the history is usually shorter.

2. A 24 year old man presents with localised spinal pain over 2 months which is worsened on
movement. He is known to be an IVDU. He has no history suggestive of tuberculosis. The pain is
now excruciating at rest and not improving with analgesia. He has a temperature of 39 oC.

You answered Subacute degeneration of the cord

The correct answer is Osteomyelitis

In an IVDU with back pain and pyrexia have a high suspicion for osteomylelitis. The most likely
organism is staph aureus and the cervical spine is the most common region affected. TB tends to
affect the thoracic spine and in other causes of osteomyelitis the lumbar spine is affected.

3. A 22 year man is shot in the back, in the lumbar region. He has increased tone and hyper-reflexia of
his right leg. He cannot feel his left leg.

Brown-Sequard syndrome

Theme from January 2012 exam


Brown -Sequard syndrome is caused by hemisection of the spinal cord. It may result from stab
injuries or lateral vertebral fractures. It results in ipsilateral paralysis (pyramidal tract) , and also loss
of proprioception and fine discrimination (dorsal columns). Pain and temperature sensation are lost
on the contra-lateral side. This is because the fibres of the spinothalamic tract have decussated below
the level of the cord transection.

Spinal disorders

Dorsal column lesion • Loss vibration and proprioception


• Tabes dorsalis, SACD

Spinothalamic tract lesion • Loss of pain, sensation and temperature

Central cord lesion • Flaccid paralysis of the upper limbs

Osteomyelitis • Normally progressive


• Staph aureus in IVDU, normally cervical region affected
• Fungal infections in immunocompromised
• Thoracic region affected in TB

Infarction spinal cord • Dorsal column signs (loss of proprioception and fine discrimination

Cord compression • UMN signs


• Malignancy
• Haematoma
• Fracture

Brown-sequard syndrome • Hemisection of the spinal cord


• Ipsilateral paralysis
• Ipsilateral loss of proprioception and fine discrimination
• Contralateral loss of pain and temperature
Image sourced from Wikipedia
Image sourced from Wikipedia

Dermatomes

• C2 to C4 The C2 dermatome covers the occiput and the top part of the neck. C3 covers the lower part
of the neck to the clavicle. C4 covers the area just below the clavicle.
• C5 to T1 Situated in the arms. C5 covers the lateral arm at and above the elbow. C6 covers the
forearm and the radial (thumb) side of the hand. C7 is the middle finger, C8 is the lateral aspects of
the hand, and T1 covers the medial side of the forearm.
• T2 to T12 The thoracic covers the axillary and chest region. T3 to T12 covers the chest and back to
the hip girdle. The nipples are situated in the middle of T4. T10 is situated at the umbilicus. T12 ends
just above the hip girdle.
• L1 to L5 The cutaneous dermatome representing the hip girdle and groin area is innervated by L1
spinal cord. L2 and 3 cover the front part of the thighs. L4 and L5 cover medial and lateral aspects of
the lower leg.
• S1 to S5 S1 covers the heel and the middle back of the leg. S2 covers the back of the thighs. S3 cover
the medial side of the buttocks and S4-5 covers the perineal region. S5 is of course the lowest
dermatome and represents the skin immediately at and adjacent to the anus.

Myotomes

Upper limb
Elbow flexors/Biceps C5
Wrist extensors C6
Elbow extensors/Triceps C7
Long finger flexors C8
Small finger abductors T1

Lower limb
Hip flexors (psoas) L1 and L2
Knee extensors (quadriceps) L3
Ankle dorsiflexors (tibialis anterior) L4 and L5
Toe extensors (hallucis longus) L5
Ankle plantar flexors (gastrocnemius) S1

The anal sphincter is innervated by S2,3,4

Question 4 of 87
A 24 year old man is brought to the emergency department have suffered a crush injury to his forearm.
Assessment demonstrates that the arm is tender, red and swollen. There is clinical evidence of an ulnar fracture
and the patient cannot move their fingers. Which is the most appropriate course of action?

A. Application of an external fixation device

B. Closed reduction

C. Debridement

D. Discharge and review in fracture clinic


E. Fasciotomy

Theme from April 2012


The combination of a crush injury, limb swelling and inability to move digits should raise suspicion of a
compartment syndrome that will require a fasciotomy

Compartment syndrome

• This is a particular complication that may occur following fractures (or following ischaemia re-
perfusion injury in vascular patients). It is characterised by raised pressure within a closed anatomical
space.
• The raised pressure within the compartment will eventually compromise tissue perfusion resulting in
necrosis. The two main fractures carrying this complication include supracondylar fractures and tibial
shaft injuries.

Symptoms and signs

• Pain, especially on movement (even passive)


• Parasthesiae
• Pallor may be present
• Arterial pulsation may still be felt as the necrosis occurs as a result of microvascular compromise
• Paralysis of the muscle group may occur

Diagnosis

• Is made by measurement of intracompartmental pressure measurements. Pressures in excess of


20mmHg are abnormal and >40mmHg is diagnostic.

Treatment

• This is essentially prompt and extensive fasciotomies


• In the lower limb the deep muscles may be inadequately decompressed by the inexperienced operator
when smaller incisions are performed
• Myoglobinuria may occur following fasciotomy and result in renal failure and for this reason these
patients require aggressive IV fluids
• Where muscle groups are frankly necrotic at fasciotomy they should be debrided and amputation may
have to be considered
• Death of muscle groups may occur within 4-6 hours

1/3 Question 1-3 of 83 Next


Theme: Disorders of the knee

A. Chondromalacia patellae
B. Dislocated patella
C. Undisplaced fracture patella
D. Displaced patella fracture
E. Avulsion fracture of the tibial tubercle
F. Quadriceps tendon rupture
G. Osgood Schlatters disease

Please select the most likely explanation for the scenario given. Each option may be used once, more than once
or not at all.

1. A 19 year old sportswoman presents with knee pain which is worse on walking down the stairs and when
sitting still. On examination there is wasting of the quadriceps and pseudolocking of the knee.

You answered Quadriceps tendon rupture

The correct answer is Chondromalacia patellae

A teenage girl with knee pain on walking down the stairs is characteristic for chondromalacia
patellae(anterior knee pain). Most cases are managed with physiotherapy.

2. A tall 18 year old male athlete is admitted to the emergency room after being hit in the knee by a hockey
stick. On examination his knee is tense and swollen. X-ray shows no fractures.

Dislocated patella

A patella dislocation is a common cause of haemarthrosis and many will spontaneously reduce when the
leg is straightened. In the chronic setting physiotherapy is used to strengthen the quadriceps muscles.

3. An athletic 15 year old boy presents with knee pain of 3 weeks duration. It is worst during activity and
settles with rest. On examination there is tenderness overlying the tibial tuberosity and an associated
swelling at this site.

You answered Avulsion fracture of the tibial tubercle

The correct answer is Osgood Schlatters disease

Athletic boys and girls may develop this condition in their teenage years. It is caused by multiple micro
fractures at the point of insertion of the tendon into the tibial tuberosity. Most cases settle with
physiotherapy and rest.

Next question
Knee injury

Types of injury

Ruptured anterior • Sport injury


cruciate ligament • Mechanism: high twisting force applied to a bent knee
• Typically presents with: loud crack, pain and RAPID swelling knee
(haemoarthrosis)
• Poor healing
• Management: intense physiotherapy or surgery

Ruptured posterior • Mechanism: hyperextension injuries


cruciate ligament • Tibia lies back on the femur
• Paradoxical anterior draw test

Rupture of medial • Mechanism: leg forced into valgus via force outside the leg
collateral ligament • Knee unstable when put into valgus position

Menisceal tear • Rotational sporting injuries


• Delayed knee swelling
• Joint locking (Patient may develop skills to "unlock" the knee
• Recurrent episodes of pain and effusions are common, often following
minor trauma

Chondromalacia • Teenage girls, following an injury to knee e.g. Dislocation patella


patellae • Typical history of pain on going downstairs or at rest
• Tenderness, quadriceps wasting

Dislocation of the • Most commonly occurs as a traumatic primary event, either through
patella direct trauma or through severe contraction of quadriceps with knee
streched in valgus and external rotation
• Genu valgum, tibial torsion and high riding patella are risk factors
• Skyline x-ray views of patella are required, although displaced patella
may be clinically obvious
• An osteochondral fracture is present in 5%
• The condition has a 20% recurrence rate

Fractured patella • 2 types:

i. Direct blow to patella causing undisplaced fragments


ii. Avulsion fracture

Tibial plateau fracture • Occur in the elderly (or following significant trauma in young)
• Mechanism: knee forced into valgus or varus, but the knee fractures
before the ligaments rupture
• Varus injury affects medial plateau and if valgus injury, lateral plateau
depressed fracture occurs
• Classified using the Schatzker system (see below)

Schatzker Classification system for tibial plateau fractures


Type Features
1- vertical split of lateral condyle Fracture through dense bone, usually in the young. It may be virtually
undisplaced, or the condylar fragment may be pushed inferiorly and tilted
2- a vertical split of the lateral The wedge fragement (which may be of variable size), is displaced
condyle combined with an laterally; the joint is widened. Untreated, a valgus deformity may develop
adjacent loadbearing part of the
condyle
3- depression of the articular The split does not extend to the edge of the plateau. Depressed fragments
surface with intact condylar rim may be firmly embedded in subchondral bone, the joint is stable
4- fragment of the medial tibial Two injuries are seen in this category; (1) a depressed fracture of
condyle osteoporotic bone in the elderly. (2) a high energy fracture resulting in a
condylar split that runs from the intercondylar eminence to the medial
cortex. Associated ligamentous injury may be severe
5-fracture of both condyles Both condyles fractured but the column of the metaphysis remains in
continuity with the tibial shaft
6-combined condylar and High energy fracture with marked comminution
subcondylar fractures

Previous Question 4 of 83 Next


A 10 year old boy presents with symptoms of right knee pain. The pain has been present on most occasions for
the past three months and the pain typically lasts for several hours at a time. On examination he walks with an
antalgic gait and has apparent right leg shortening. What is the most likely diagnosis?
A. Perthes Disease

B. Osteosarcoma of the femur

C. Osteoarthritis of the hip

D. Transient synovitis of the hip

E. Torn medial meniscus

Next question

Theme from September 2012 Exam


There are many causes of the irritable hip in the 10-14 year age group. Many of these may cause both hip pain
or knee pain. Transient synovitis of the hip the commonest disorder but does not typically last for 3 months.
An osteosarcoma would not usually present with apparent limb shortening unless pathological fracture had
occurred. A slipped upper femoral epiphysis can cause a similar presentation although it typically presents
later and with different patient characteristics.

Perthes disease

Perthes disease

• Idiopathic avascular necrosis of the femoral epiphysis of the femoral head


• Impaired blood supply to femoral head, causing bone infarction. New vessels develop and ossification
occurs. The bone either heals or a subchondral fracture occurs.

Clinical features

• Males 4x's greater than females


• Age between 2-12 years (the younger the age of onset, the better the prognosis)
• Limp
• Hip pain
• Bilateral in 20%

Diagnosis
Plain x-ray, Technetium bone scan or magnetic resonance imaging if normal x-ray and symptoms persist.

Catterall staging
Stage Features

Stage 1 Clinical and histological features only

Stage 2 Sclerosis with or without cystic changes and preservation of the articular surface

Stage 3 Loss of structural integrity of the femoral head

Stage 4 Loss of acetabular integrity

Management

• To keep the femoral head within the acetabulum: cast, braces


• If less than 6 years: observation
• Older: surgical management with moderate results
• Operate on severe deformities

Prognosis
Most cases will resolve with conservative management. Early diagnosis improves outcomes.

Previous Question 5 of 83 Next


Which of the following types of growth plate fractures may have similar radiological appearances?

A. Salter Harris types 1 and 5

B. Salter Harris types 4 and 5

C. Salter Harris types 3 and 5

D. Salter Harris types 1 and 2

E. Salter Harris types 1 and 3

Next question

Salter Harris injury types 1 and 5 (transverse fracture through growth plate Vs. Compression fracture) may
mimic each other radiologically. Type 5 injuries have the worst outcomes. Radiological signs of type 5 injuries
are subtle and may include narrowing of the growth plate.
Epiphyseal fractures

Fractures involving the growth plate in children are classified using the Salter - Harris system.
There are 5 main types.

Salter Harris Classification

Type Description

Type 1 Transverse fracture through the growth plate

Type 2 Fracture through the growth plate to the metaphysis (commonest type)

Type 3 Fracture through the growth plate and the epiphysis with metaphysis spared

Type 4 Fracture involving the growth plate, metaphysis and epiphysis

Type 5 Compression fracture of the growth plate (worst outcome)

Management
Non displaced type 1 injuries can generally be managed conservatively. Unstable or more extensive injuries
will usually require surgical reduction and/ or fixation, as proper alignment is crucial.

Previous 3 / 3 Question 6-8 of 83 Next


Theme: Pathological fractures

A. Osteosarcoma
B. Osteomalacia
C. Osteoporosis
D. Metastatic carcinoma
E. Osteoblastoma
F. Giant cell tumour
G. Ewing's sarcoma

For each pathological fracture please select the most likely aetiology for the scenario given. Each option may
be used once, more than once or not at all.
6. A 30 year old woman presents with pain and swelling of the left shoulder. There is a large radiolucent
lesion in the head of the humerus extending to the subchondral plate.

Giant cell tumour

Giant cell tumours on x-ray have a 'soap bubble' appearance. They present as pain or pathological
fractures. They commonly metastasize to the lungs.

7. A 72 year old woman has a lumbar vertebral crush fracture. She has hypocalcaemia and a low urinary
calcium.

Osteomalacia

Hypocalcemia and low urinary calcium are biochemical features of osteomalacia. Unfortunately
surgeons do need to look at some blood results!

8. A 16 year old boy presents with severe groin pain after kicking a football. Imaging confirms a pelvic
fracture. A previous pelvic x-ray performed 2 weeks ago shows a lytic lesion with 'onion type' periosteal
reaction.

Ewing's sarcoma

A Ewings sarcoma is most common in males between 10-20 years. It can occur in girls. A lytic lesion
with a lamellated or onion type periosteal reaction is a classical finding on x-rays. Most patients present
with metastatic disease with a 5 year prognosis between 5-10%.

Next question

Pathological fractures

• A pathological fracture occurs in abnormal bone due to insignificant injury

Causes

Metastatic tumours • Breast


• Lung
• Thyroid
• Renal
• Prostate

Bone disease • Osteogenesis imperfecta


• Osteoporosis
• Metabolic bone disease
• Paget's disease

Local benign conditions • Chronic osteomyelitis


• Solitary bone cyst

Primary malignant tumours • Chondrosarcoma


• Osteosarcoma
• Ewing's tumour

Previous Question 9 of 83 Next


An 8 year old child falls onto an outstretched hand and is brought to the emergency department. He is
examined by a doctor and a bony injury is cleared clinically. He re-presents a week later with pain in his hand.
What is the most likely underlying injury?

A. Fracture of the distal radius

B. Fracture of the scaphoid

C. Dislocation of the lunate

D. Rupture of flexor pollicis longus tendon

E. Bennett's fracture

Next question

Theme from January 2013 Exam


Scaphoid fractures in children will usually involve the distal pole and are easily missed. The initial clinical
examination (and sometimes x-rays) may be normal and repeated clinical examination and imaging is advised
for this reason. Whilst the other injuries may be sustained from a fall onto an outstretched hand they are less
likely to be overlooked on clinical examination. In the case of a Bennetts fracture, the injury mechanism is less
compatible with this type of injury.

Scaphoid fractures

• Scaphoid fractures are the commonest carpal fractures.


• Surface of scaphoid is covered by articular cartilage with small area available for blood vessels
(fracture risks blood supply)
• Forms floor of anatomical snuffbox
• Risk of fracture associated with fall onto outstretched hand (tubercle, waist, or proximal third)
• Ulnar deviation AP needed for visualization of scaphoid
• Immobilization of scaphoid fractures difficult

Management

Non-displaced fractures - Casts or splints


- Percutaneous scaphoid fixation

Displaced fracture Surgical fixation, usually with a screw

Complications

• Non union of scaphoid


• Avascular necrosis of the scaphoid
• Scapholunate disruption and wrist collapse
• Degenerative changes of the adjacent joint

Previous 2 / 3 Question 10-12 of 83 Next


Theme: Shoulder injuries

A. Glenohumeral dislocation
B. Acromioclavicular dislocation
C. Sternoclavicular dislocation
D. Biceps tendon tear
E. Supraspinatus tear
F. Fracture of the surgical neck of the humerus
G. Infra spinatus tear

For each scenario please select the most likely underlying diagnosis. Each option may be used once, more than
once or not at all.

10. A 23 year old rugby player falls directly onto his shoulder. There is pain and swelling of the shoulder
joint. The clavicle is prominent and there appears to be a step deformity.

Acromioclavicular dislocation

Acromioclavicular joint (ACJ) dislocation normally occurs secondary to direct injury to the superior
aspect of the acromion. Loss of shoulder contour and prominent classical are key features. Note; rotator
cuff tears rarely occur in the second decade.

11. A 22 year old man falls over and presents to casualty. A shoulder x-ray is performed, the radiologist
comments that a Hill-Sachs lesion is present.

You answered Biceps tendon tear

The correct answer is Glenohumeral dislocation

A Hill-Sachs lesion is when the cartilage surface of the humerus is in contact with the rim of the
glenoid. About 50% of anterior glenohumeral dislocations are associated with this lesion.

12. An 82 year old female presents to A&E after tripping on a step. She complains of shoulder pain. On
examination there is pain to 90o on abduction.

Supraspinatus tear

A supraspinatus tear is the most common of rotator cuff tears. It occurs as a result of degeneration and
is rare in younger adults.

Next question

Shoulder disorders

Proximal humerus fractures


• Very common. Usually through the surgical neck. Number of classification systems though for
practical purposes describing the number of fracture fragments is probably easier. Some key points:
• It is rare to have fractures through the anatomical neck.
• Anatomical neck fractures which are displaced by >1cm carry a risk of avascular necrosis to the
humeral head.
• In children the commonest injury pattern is a greenstick fracture through the surgical neck.
• Impacted fractures of the surgical neck are usually managed with a collar and cuff for 3 weeks
followed by physiotherapy.
• More significant displaced fractures may require open reduction and fixation or use of an
intramedullary device.

Types of shoulder dislocation

• Glenohumeral dislocation (commonest): anterior shoulder dislocation most common


• Acromioclavicular dislocation (12%): clavicle loses all attachment with the scapula
• Sternoclavicular dislocation (uncommon)

Types of glenohumeral dislocation:

Anterior shoulder • External rotation and abduction


dislocation • 35-40% recurrent (it is the commonest disorder)
• Associated with greater tuberosity fracture, Bankart lesion, Hill-Sachs
defect

Inferior shoulder Luxatio erecta


dislocation

Posterior shoulder • Proportion misdiagnosed.


dislocation • Rim's sign, light bulb sign.
• Associated with Trough sign

Superior shoulder Rare and usually follow major trauma.


dislocation

Treatment
Prompt reduction is the mainstay of treatment and is usually performed in the emergency department.
Neurovascular status must be checked pre and post reduction and x-rays should be performed again post
reduction to ensure no fracture has occurred. In recurrent anterior dislocation there is usually a Bankart lesion
and this may be repaired surgically. Recurrent posterior dislocations may be repaired in a similar manner to
anterior lesions but using a posterior (or arthroscopic) approach.

Previous Question 13 of 83 Next


Which of the following statements relating to menisceal tears is false?

A. The medial meniscus is most often affected

B. True locking of the knee joint may occur

C. Most established tears will heal with conservative management

D. In the chronic setting there is typically little to find on examination if the knee
is not locked

E. An arthroscopic approach may be used to treat most lesions

Next question

Menisci have no nerve or blood supply and thus heal poorly. Established tears with associated symptoms are
best managed by arthroscopic menisectomy.

Knee injury

Types of injury

Ruptured anterior • Sport injury


cruciate ligament • Mechanism: high twisting force applied to a bent knee
• Typically presents with: loud crack, pain and RAPID swelling knee
(haemoarthrosis)
• Poor healing
• Management: intense physiotherapy or surgery

Ruptured posterior • Mechanism: hyperextension injuries


cruciate ligament • Tibia lies back on the femur
• Paradoxical anterior draw test

Rupture of medial • Mechanism: leg forced into valgus via force outside the leg
collateral ligament
• Knee unstable when put into valgus position

Menisceal tear • Rotational sporting injuries


• Delayed knee swelling
• Joint locking (Patient may develop skills to "unlock" the knee
• Recurrent episodes of pain and effusions are common, often following
minor trauma

Chondromalacia • Teenage girls, following an injury to knee e.g. Dislocation patella


patellae • Typical history of pain on going downstairs or at rest
• Tenderness, quadriceps wasting

Dislocation of the • Most commonly occurs as a traumatic primary event, either through
patella direct trauma or through severe contraction of quadriceps with knee
streched in valgus and external rotation
• Genu valgum, tibial torsion and high riding patella are risk factors
• Skyline x-ray views of patella are required, although displaced patella
may be clinically obvious
• An osteochondral fracture is present in 5%
• The condition has a 20% recurrence rate

Fractured patella • 2 types:

i. Direct blow to patella causing undisplaced fragments


ii. Avulsion fracture

Tibial plateau fracture • Occur in the elderly (or following significant trauma in young)
• Mechanism: knee forced into valgus or varus, but the knee fractures
before the ligaments rupture
• Varus injury affects medial plateau and if valgus injury, lateral plateau
depressed fracture occurs
• Classified using the Schatzker system (see below)

Schatzker Classification system for tibial plateau fractures


Type Features
1- vertical split of lateral condyle Fracture through dense bone, usually in the young. It may be virtually
undisplaced, or the condylar fragment may be pushed inferiorly and tilted
2- a vertical split of the lateral The wedge fragement (which may be of variable size), is displaced
condyle combined with an laterally; the joint is widened. Untreated, a valgus deformity may develop
adjacent loadbearing part of the
condyle
3- depression of the articular The split does not extend to the edge of the plateau. Depressed fragments
surface with intact condylar rim may be firmly embedded in subchondral bone, the joint is stable
4- fragment of the medial tibial Two injuries are seen in this category; (1) a depressed fracture of
condyle osteoporotic bone in the elderly. (2) a high energy fracture resulting in a
condylar split that runs from the intercondylar eminence to the medial
cortex. Associated ligamentous injury may be severe
5-fracture of both condyles Both condyles fractured but the column of the metaphysis remains in
continuity with the tibial shaft
6-combined condylar and High energy fracture with marked comminution
subcondylar fractures

Previous 0 / 3 Question 14-16 of 83 Next


Theme: Developmental bone disorders

A. Rickets
B. Craniocleidodysostosis
C. Achondroplasia
D. Scurvy
E. Pagets disease
F. Multiple myeloma
G. Osteogenesis imperfecta
H. Osteomalacia
I. Osteopetrosis
J. None of the above

Please select the most likely disease process to account for the clinical scenario. Each option may be used
once, more than once or not at all

14. A 15 year-old boy presents to the out-patient clinic with tiredness, recurrent throat and chest infections,
and gradual loss of vision. Multiple x-rays show brittle bones with no differentiation between the cortex
and the medulla.

You answered Scurvy


The correct answer is Osteopetrosis

Osteopetrosis is an autosomal recessive condition. It is commonest in young adults. They may present
with symptoms of anaemia or thrombocytopaenia due to decreased marrow space. Radiology reveals a
lack of differentiation between the cortex and the medulla described as marble bone. These bones are
very dense and brittle.

15. A 12 year-old boy who is small for his age presents to the clinic with poor muscular development and
hyper-mobile fingers. His x rays show multiple fractures of the long bones and irregular patches of
ossification.

You answered Rickets

The correct answer is Osteogenesis imperfecta

Osteogenesis imperfecta is caused by defective osteoid formation due to congenital inability to produce
adequate intercellular substances like osteoid, collagen and dentine. There is a failure of maturation of
collagen in all the connective tissues.Radiology may show translucent bones, multiple fractures,
particularly of the long bones, wormian bones (irregular patches of ossification) and a trefoil pelvis.

16. A 1 year-old is brought to the Emergency Department with a history of failure to thrive. On
examination, the child is small for age and has a large head. X-ray shows a cupped appearance of the
epiphysis of the wrist.

You answered Achondroplasia

The correct answer is Rickets

Rickets is the childhood form of osteomalacia. It is due to the failure of the osteoid to ossify due to
vitamin D deficiency. Symptoms start about the age of one. The child is small for age and there is a
history of failure to thrive. Bony deformities include bowing of the femur and tibia, a large head,
deformity of the chest wall with thickening of the costochondral junction (ricketty rosary), and a
transverse sulcus in the chest caused by the pull of the diaphragm (Harrison's sulcus). X- Rays show
widening and cupping of the epiphysis of the long bones, most readily apparent in the wrist.

Next question

Paediatric fractures

Paediatric fracture types


Type Injury pattern

Complete fracture Both sides of cortex are breached

Toddlers fracture Oblique tibial fracture in infants

Plastic deformity Stress on bone resulting in deformity without cortical disruption

Greenstick fracture Unilateral cortical breach only

Buckle fracture Incomplete cortical disruption resulting in periosteal haematoma only

Growth plate fractures


In paediatric practice fractures may also involve the growth plate and these injuries are classified according to
the Salter- Harris system (given below):

Type Injury pattern

I Fracture through the physis only (x-ray often normal)

II Fracture through the physis and metaphysis

III Fracture through the physis and epiphyisis to include the joint

IV Fracture involving the physis, metaphysis and epiphysis

V Crush injury involving the physis (x-ray may resemble type I, and appear normal)

As a general rule it is safer to assume that growth plate tenderness is indicative of an underlying fracture even
if the x-ray appears normal. Injuries of Types III, IV and V will usually require surgery. Type V injuries are
often associated with disruption to growth.

Non accidental injury

• Delayed presentation
• Delay in attaining milestones
• Lack of concordance between proposed and actual mechanism of injury
• Multiple injuries
• Injuries at sites not commonly exposed to trauma
• Children on the at risk register

Pathological fractures
Genetic conditions, such as osteogenesis imperfecta, may cause pathological fractures.

Osteogenesis imperfecta

• Defective osteoid formation due to congenital inability to produce adequate intercellular substances
like osteoid, collagen and dentine.
• Failure of maturation of collagen in all the connective tissues.
• Radiology may show translucent bones, multiple fractures, particularly of the long bones, wormian
bones (irregular patches of ossification) and a trefoil pelvis.

Subtypes

• Type I The collagen is normal quality but insufficient quantity.


• Type II- Poor collagen quantity and quality.
• Type III- Collagen poorly formed. Normal quantity.
• Type IV- Sufficient collagen quantity but poor quality.

Osteopetrosis

• Bones become harder and more dense.


• Autosomal recessive condition.
• It is commonest in young adults.
• Radiology reveals a lack of differentiation between the cortex and the medulla described as marble
bone.

Previous 0 / 3 Question 17-19 of 83 Next


Theme: Hip fractures

A. Conservative management
B. Percutaneous pinning
C. Fracture reduction and internal fixation
D. Hemiarthroplasty
E. Total hip replacement
F. Dynamic hip screw
G. Intramedullary femoral nail

For each scenario please select the most appropriate management option. Each option may be used once, more
than once or not at all.

17. A 60 year old male is admitted to A&E with a fall. He lives with his wife and still works as a restaurant
manager. He has a past history of benign prostatic hypertrophy and is currently taking tamsulosin. He is
otherwise fit and healthy. On examination there is right hip tenderness on movement in all directions. A
hip x-ray confirms an intertrochanteric fracture.

You answered Intramedullary femoral nail

The correct answer is Dynamic hip screw

The blood supply to the femoral head may be intact and the fracture should heal with compression type
devices such as gamma nails or dynamic hip screws. The latter device being the most commonly
performed therapeutic intervention.

18. An 86 year old retired pharmacist is admitted to A&E following a fall. She complains of right hip pain.
She is known to have hypertension and is currently on bendrofluazide. She lives alone and mobilises
with a Zimmer frame. Her right leg is shortened and externally rotated. A hip x-ray confirms a
displaced intracapsular fracture.

You answered Total hip replacement

The correct answer is Hemiarthroplasty

Hemiarthroplasty is offered to older, less mobile individuals compared to fracture reduction and
fixation in younger patients.

19. A 74 year old male is admitted to A&E with a fall. He is known to have rheumatoid arthritis and is on
methotrexate and paracetamol. He lives alone in a bungalow and enjoys playing golf. He is independent
with his ADLs. He complains of left groin pain, therefore has a hip x-ray which confirms a displaced
intracapsular fracture.

You answered Conservative management

The correct answer is Total hip replacement

This patient has pre-existing joint disease, good level of activity and a relatively high life expectancy,
therefore THR is preferable to hemiarthroplasty.
Next question

Hip fractures

The hip is a common site of fracture especially in osteoporotic, elderly females. The blood supply to the
femoral head runs up the neck and thus avascular necrosis is a risk in displaced fractures.

Classification
The Garden system is one classification system in common use.

• Type I: Stable fracture with impaction in valgus.


• Type II: Complete fracture but undisplaced.
• Type III: Displaced fracture, usually rotated and angulated, but still has bony contact.
• Type IV: Complete bony disruption.

Blood supply disruption is most common following Types III and IV.

Management of hip fractures in older adults


SIGN Guidelines

Fracture type Patient co-morbidities Management

Undisplaced intracapsular Nil Internal fixation (especially if young)


fracture

Undisplaced intracapsular Major illness or advanced Hemiarthroplasty


fracture organ specific disease

Displaced intracapsular Nil If age <70 then internal fixation (if


fracture possible), hip arthroplasty if not

Displaced intracapsular Nil Age >70 total hip arthroplasty


fracture

Displaced intracapsular Major/ immobile Hemiarthroplasty


fracture

Extracapsular fracture (non Only major co-morbidities Dynamic hip screw


special type) affect management
Extracapsular fracture Only major co-morbidities Usually intramedullary device
(reverse oblique, transverse or affect management
sub trochanteric)

A typical image of an intracapsular fracture occurring in an elderly osteoporotic lady

Image sourced from Wikipedia

Question 20
Previous Next
of 83
Of the list below, which is not a cause of avascular necrosis?

A. Steroids

B. Sickle cell disease

C. Radiotherapy

D. Myeloma

E. Caisson disease

Next question
Causes of avascular necrosis
P ancreatitis
L upus
A lcohol
S teroids
T rauma
I diopathic, infection
C aisson disease, collagen vascular disease
R adiation, rheumatoid arthritis
A myloid
G aucher disease
S ickle cell disease

Steroid containing therapy for myeloma may induce avascular necrosis, however the disease itself does not
cause it. Caisson disease as may occur in deep sea divers is a recognised cause.

Avascular necrosis

• Cellular death of bone components due to interruption of the blood supply, causing bone destruction
• Main joints affected are hip, scaphoid, lunate and the talus.
• It is not the same as non union. The fracture has usually united.
• Radiological evidence is slow to appear.
• Vascular ingrowth into the affected bone may occur. However, many joints will develop secondary
osteoarthritis.

Causes
P ancreatitis
L upus
A lcohol
S teroids
T rauma
I diopathic, infection
C aisson disease, collagen vascular disease
R adiation, rheumatoid arthritis
A myloid
G aucher disease
S ickle cell disease
Presentation
Usually pain. Often despite apparent fracture union.

Investigation
MRI scanning will show changes earlier than plain films.

Treatment
In fractures at high risk sites anticipation is key. Early prompt and accurate reduction is essential.

Non weight bearing may help to facilitate vascular regeneration.

Joint replacement may be necessary, or even the preferred option (e.g. Hip in the elderly).

Previous Question 21 of 83 Next


Which of the following is the first radiological change likely to be apparent in a plain radiograph of a 12 year
old presenting with suspected Perthes disease

A. Multiple bone cysts

B. Sclerosis of the femoral head

C. Loss of bone density

D. Joint space narrowing

E. Collapse of the femoral head

Next question

In Catterall stage I disease there may be no radiological abnormality at all. In Stage II disease there may be
sclerosis of the femoral head.

Indication for treatment (aide memoire):Half a dozen, half a head


Those aged greater than 6 years with >50% involvement of the femoral head should almost always be treated.

Previous 0 / 3 Question 22-24 of 83 Next


Theme: Upper limb injuries
A. Pulled elbow
B. Fracture of the coronoid process
C. Scaphoid fracture
D. Fracture of the distal humerus
E. Bennets fracture
F. Fracture of the shaft of the radius and ulnar
G. Galeazzi fracture
H. Fracture of the olecranon
I. Fracture of the radial head

Please select the most likely injury for the scenario given. Each option may be used once, more than once or
not at all.

22. A 32 year old man presents with a painful swelling over the volar aspect of his hand after receiving a
hard blow to his palm. On examination, he experiences pain on moving the wrist and on longitudinal
compression of the thumb.

You answered Galeazzi fracture

The correct answer is Scaphoid fracture

Scaphoid fractures usually occur as a result of direct hard blow to the palm or following a fall on the
out-stretched hand. The main physical signs are swelling and tenderness in the anatomical snuff box,
and pain on wrist movements and on longitudinal compression of the thumb

23. A 26 year old man presents to the emergency department with a swelling over his left elbow after a fall
on an outstretched hand. On examination, he has tenderness over the proximal part of his forearm, and
has severely restricted supination and pronation movements.

You answered Fracture of the olecranon

The correct answer is Fracture of the radial head

Fracture of the radial head is common in young adults. It is usually caused by a fall on the outstretched
hand. On examination, there is marked local tenderness over the head of the radius, impaired
movements at the elbow, and a sharp pain at the lateral side of the elbow at the extremes of rotation
(pronation and supination).
24. A 56 year old lady presents with a painful swelling over the lower end of the forearm following a fall.
Imaging reveals a distal radial fracture with disruption of the distal radio-ulnar joint.

You answered Bennets fracture

The correct answer is Galeazzi fracture

Galeazzi fractures occur after a fall on the hand with a rotational force superimposed on it. On
examination, there is bruising, swelling and tenderness over the lower end of the forearm. X- Rays
reveal a displaced fracture of the radius and a prominent ulnar head due to dislocation of the inferior
radio-ulnar joint.

Next question

Upper limb fractures

Colles' fracture

• Fall onto extended outstretched hands


• Described as a dinner fork type deformity
• Classical Colles' fractures have the following 3 features:

Features of the injury


1. Transverse fracture of the radius
2. 1 inch proximal to the radio-carpal joint
3. Dorsal displacement and angulation

Smith's fracture (reverse Colles' fracture)

• Volar angulation of distal radius fragment (Garden spade deformity)


• Caused by falling backwards onto the palm of an outstretched hand or falling with wrists flexed

Bennett's fracture

• Intra-articular fracture of the first carpometacarpal joint


• Impact on flexed metacarpal, caused by fist fights
• X-ray: triangular fragment at ulnar base of metacarpal
Monteggia's fracture

• Dislocation of the proximal radioulnar joint in association with an ulna fracture


• Fall on outstretched hand with forced pronation
• Needs prompt diagnosis to avoid disability

Galeazzi fracture

• Radial shaft fracture with associated dislocation of the distal radioulnar joint
• Occur after a fall on the hand with a rotational force superimposed on it.
• On examination, there is bruising, swelling and tenderness over the lower end of the forearm.
• X Rays reveal the displaced fracture of the radius and a prominent ulnar head due to dislocation of the
inferior radio-ulnar joint.

Barton's fracture

• Distal radius fracture (Colles'/Smith's) with associated radiocarpal dislocation


• Fall onto extended and pronated wrist

Scaphoid fractures

• Scaphoid fractures are the commonest carpal fractures.


• Surface of scaphoid is covered by articular cartilage with small area available for blood vessels
(fracture risks blood supply)
• Forms floor of anatomical snuffbox
• Risk of fracture associated with fall onto outstretched hand (tubercle, waist, or proximal 1/3)
• The main physical signs are swelling and tenderness in the anatomical snuff box, and pain on wrist
movements and on longitudinal compression of the thumb.
• Ulnar deviation AP needed for visualization of scaphoid
• Immobilization of scaphoid fractures difficult

Radial head fracture

• Fracture of the radial head is common in young adults.


• It is usually caused by a fall on the outstretched hand.
• On examination, there is marked local tenderness over the head of the radius, impaired movements at
the elbow, and a sharp pain at the lateral side of the elbow at the extremes of rotation (pronation and
supination).

Previous 2 / 3 Question 25-27 of 83 Next


Theme: Hand injuries

A. Admission and surgical debridement


B. Application of futura splint and fracture clinic review
C. Application of tubigrip bandage and fracture clinic review
D. Admission for open reduction and fixation
E. Discharge with reassurance
F. Commence oral prednisolone
G. Commence oral diclofenac

Which of the following options is the best management plan? Each option may be used once, more than once
or not at all.

25. A 42 year old skier falls and impacts his hand on his ski pole. On examination he is tender in the
anatomical snuffbox and on bimanual palpation. Xrays with scaphoid views show no evidence of
fracture.

Application of futura splint and fracture clinic review

A fracture may still be present and should be immobilised until repeat imaging can be performed.

26. A 43 year old man falls over landing on his left hand. Although there was anatomical snuffbox
tenderness no x-rays either at the time or subsequently have shown evidence of scaphoid fracture. He
has been immobilised in a futura splint for two weeks and is now asymptomatic.

Discharge with reassurance

This patient is at extremely low risk of having sustained a scaphoid injury and may be discharged.

27. A builder falls from scaffolding and lands on his left hand he suffers a severe laceration to his palm. An
x-ray shows evidence of scaphoid fracture that is minimally displaced.

You answered Admission for open reduction and fixation


The correct answer is Admission and surgical debridement

This is technically an open fracture and should be debrided prior to attempted fixation (which should
occur soon after).

Next question

Scaphoid fractures:
80% of all carpal fractures
80% occur in men
80% occur at the waist of the scaphoid

Scaphoid fractures

• Scaphoid fractures are the commonest carpal fractures.


• Surface of scaphoid is covered by articular cartilage with small area available for blood vessels
(fracture risks blood supply)
• Forms floor of anatomical snuffbox
• Risk of fracture associated with fall onto outstretched hand (tubercle, waist, or proximal third)
• Ulnar deviation AP needed for visualization of scaphoid
• Immobilization of scaphoid fractures difficult

Management

Non-displaced fractures - Casts or splints


- Percutaneous scaphoid fixation

Displaced fracture Surgical fixation, usually with a screw

Complications

• Non union of scaphoid


• Avascular necrosis of the scaphoid
• Scapholunate disruption and wrist collapse
• Degenerative changes of the adjacent joint
• Previous 3/3 Question 28-30 of 83 Next
Theme: Paediatric orthopaedics

A. Musculoskeletal pain
B. Congenital dysplasia of the hip
C. Slipped upper femoral epiphysis
D. Transient synovitis
E. Septic arthritis
F. Perthes disease
G. Tibial fracture

Please select the most likely diagnosis for the scenario given. Each option may be used once, more than once
or not at all.

28. A 4 year boy presents with an abnormal gait. He has a history of recent viral illness. His WCC is 11 and
ESR is 30.

Transient synovitis

Viral illnesses can be associated with transient synovitis. The WCC should ideally be > 12 and the ESR
> 40 to suggest septic arthritis.

29. A 6 year old boy presents with an groin pain. He is known to be disruptive in class. He reports that he is
bullied for being short. On examination he has an antalgic gait and pain on internal rotation of the right
hip.

Perthes disease

This child is short, has hyperactivity (disruptive behaviour) and is within the age range for Perthes
disease. Hyperactivity and short stature are associated with Perthes disease.

30. An obese 12 year old boy is referred with pain in the left knee and hip. On examination he has an
antaglic gait and limitation of internal rotation. His knee has normal range of passive and active
movement.

Slipped upper femoral epiphysis

Similar theme to September 2012 Exam


Slipped upper femoral epiphysis is commonest in obese adolescent males. The x-ray will show
displacement of the femoral epiphysis inferolaterally. Treatment is usually with rest and non weight
bearing crutches.

Next question

Beware of attributing gait disorders to benign processes in young children without careful clinical and
radiological assessment.

Paediatric orthopaedics

Diagnosis Mode of presentation Treatment Radiology

Developmental Usually diagnosed in Splints and harnesses or Initially no obvious


dysplasia of the infancy by screening tests. traction. In later years change on plain films
hip May be bilateral, when osteotomy and hip and USS gives best
disease is unilateral there realignment procedures may resolution until 3
may be leg length be needed. In arthritis a joint months of age. On
inequality. As disease replacement may be needed. plain films Shentons
progresses child may limp However, this is best deferred line should form a
and then early onset if possible as it will almost smooth arc
arthritis. More common in certainly require revision
extended breech babies.

Perthes Disease Hip pain (may be referred Remove pressure from joint X-rays will show
to the knee) usually to allow normal flattened femoral head.
occurring between 5 and 12 development. Physiotherapy. Eventually in untreated
years of age. Bilateral Usually self-limiting if cases the femoral head
disease in 20%. diagnosed and treated will fragment.
promptly.

Slipped upper Typically seen in obese Bed rest and non-weight X-rays will show the
femoral male adolescents. Pain is bearing. Aim to avoid femoral head displaced
epiphysis often referred to the knee. avascular necrosis. If severe and falling
Limitation to internal slippage or risk of it inferolaterally (like a
rotation is usually seen. occurring then percutaneous melting ice cream
Knee pain is usually present pinning of the hip may be cone) The Southwick
2 months prior to hip required. angle gives indication
slipping. Bilateral in 20%. of disease severity
Previous 3 / 3 Question 31-33 of 83 Next
Theme: Eponymous fractures

A. Smith's
B. Bennett's
C. Monteggia's
D. Colle's
E. Galeazzi
F. Pott's
G. Barton's

Link the most appropriate eponymously named fracture to the scenario described. Each scenario may be used
once, more than once or not at all.

31. A 28 year old man falls on the back of his hand. On x-ray the he has a fractured distal radius
demonstrating volar displacement of the fracture.

Smith's

This is a Smith fracture (reverse Colle's fracture); unlike a Colle's this is a high velocity injury and may
require surgical correction. Note that Colles fractures are usually dorsally displaced

32. A 38 year old window cleaner falls from his ladder. He lands on his left arm and notices an obvious
injury. An x-ray and clinical examination demonstrate that has a fracture of the proximal ulna and
associated radial dislocation

Monteggia's

This constellation of injuries is referred to as a Monteggia's fracture

33. A 32 year old man falls from scaffolding and sustains an injury to his forearm. Clinical examination and
x-ray shows that he has sustained a radial fracture with dislocation of the inferior radio-ulna joint

Galeazzi

Isolated fracture of the radius alone can occur but is rare. Always check for associated injury

Next question
Eponymous fractures

Colles' fracture (dinner fork deformity)

• Fall onto extended outstretched hand


• Classical Colles' fractures have the following 3 features:

1. Transverse fracture of the radius


2. 1 inch proximal to the radio-carpal joint
3. Dorsal displacement and angulation

Smith's fracture (reverse Colles' fracture)

• Volar angulation of distal radius fragment (Garden spade deformity)


• Caused by falling backwards onto the palm of an outstretched hand or falling with wrists flexed

Bennett's fracture

• Intra-articular fracture of the first carpometacarpal joint


• Impact on flexed metacarpal, caused by fist fights
• X-ray: triangular fragment at ulnar base of metacarpal
Image sourced from Wikipedia

Monteggia's fracture

• Dislocation of the proximal radioulnar joint in association with an ulna fracture


• Fall on outstretched hand with forced pronation
• Needs prompt diagnosis to avoid disability
Image sourced from Wikipedia

Galeazzi fracture

• Radial shaft fracture with associated dislocation of the distal radioulnar joint
• Direct blow

Pott's fracture

• Bimalleolar ankle fracture


• Forced foot eversion

Barton's fracture

• Distal radius fracture (Colles'/Smith's) with associated radiocarpal dislocation


• Fall onto extended and pronated wrist

Previous Question 34 of 83 Next


A 54-year-old man presents to the Emergency Department with a 2 day history of a swollen, painful left knee.
You aspirate the joint to avoid admission to the orthopaedic wards. Aspirated joint fluid shows calcium
pyrophosphate crystals. Which of the following blood tests is most useful in revealing an underlying cause?

A. Transferrin saturation

B. ACTH

C. ANA

D. Serum ferritin

E. LDH

Next question

This is a typical presentation of pseudogout. An elevated transferrin saturation may indicate


haemochromatosis, a recognised cause of pseudogout.

A high ferritin level is also seen in haemochromatosis but can be raised in a variety of infective and
inflammatory processes, including pseudogout, as part of an acute phase response.

Pseudogout

Pseudogout is a form of microcrystal synovitis caused by the deposition of calcium pyrophosphate dihydrate in
the synovium

Risk factors

• hyperparathyroidism
• hypothyroidism
• haemochromatosis
• acromegaly
• low magnesium, low phosphate
• Wilson's disease

Features

• knee, wrist and shoulders most commonly affected


• joint aspiration: weakly-positively birefringent rhomboid shaped crystals
• x-ray: chondrocalcinosis

Management

• aspiration of joint fluid, to exclude septic arthritis


• NSAIDs or intra-articular, intra-muscular or oral steroids as for gout

Question 35 of 83 Next
A 19 year old soldier has just returned from a prolonged marching exercise and presents with a sudden onset,
severe pain, in the forefoot. Clinical examination reveals tenderness along the second metatarsal. Plain x-rays
are taken of the area, these demonstrate callus surrounding the shaft of the second metatarsal. What is the most
likely diagnosis?

A. Stress fracture

B. Mortons neuroma

C. Osteochondroma

D. Acute osteomyelitis

E. Freiberg's disease

Next question

Theme from 2011 Exam


A short history of pain together with clinical examination and radiological signs affecting the second
metatarsal favour a stress fracture. The fact that callus is present suggests that immobilisation is unlikely to be
beneficial. Freibergs disease is an anterior metatarsalgia affecting the head of the second metarsal, it typically
occurs in the pubertal growth spurt. The initial injury was thought to be due to stress microfractures at the
growth plate. The key feature in the history which distinguishes the injury as being stress fracture is the
radiology. In Freibergs disease the x-ray changes include; joint space widening, formation of bony spurs,
sclerosis and flattening of the metatarsal head.

Stress fractures

Repetitive activity and loading of normal bone may result in small hairline fractures. Whilst these may be
painful they are seldom displaced. Surrounding soft tissue injury is unusual. They may present late following
the injury, in which case callus formation may be identified on radiographs. Such cases may not require formal
immobilisation, injuries associated with severe pain and presenting at an earlier stage may benefit from
immobilisation tailored to the site of injury.

Previous Question 36 of 83 Next


A 65-year-old Asian female presents with an extracapsular neck of femur fracture. Investigations show:

Calcium 2.07 mmol/l (2.20-2.60 mmol/l)


Phosphate 0.66 mmol/l (0.8-1.40 mmol/l)
ALP 256 IU/l (44-147 IU/l)

What is the most likely diagnosis?

A. Bone tuberculosis

B. Hypoparathyroidism

C. Myeloma

D. Osteomalacia

E. Paget's disease

Next question

Osteomalacia

• low: calcium, phosphate


• raised: alkaline phosphatase

The low calcium and phosphate combined with the raised alkaline phosphatase point towards osteomalacia.

Osteomalacia

Basics

• normal bony tissue but decreased mineral content


• rickets if when growing
• osteomalacia if after epiphysis fusion
Types

• vitamin D deficiency e.g. malabsorption, lack of sunlight, diet


• renal failure
• drug induced e.g. anticonvulsants
• vitamin D resistant; inherited
• liver disease, e.g. cirrhosis

Features

• rickets: knock-knee, bow leg, features of hypocalcaemia


• osteomalacia: bone pain, fractures, muscle tenderness, proximal myopathy

Investigation

• low calcium, phosphate, 25(OH) vitamin D


• raised alkaline phosphatase
• x-ray: children - cupped, ragged metaphyseal surfaces; adults - translucent bands (Looser's zones or
pseudofractures)

Treatment

• calcium with vitamin D tablets

Previous Question 37 of 83 Next


A 78-year-old woman is discharged following a fractured neck of femur. On review she is making good
progress but consideration is given to secondary prevention of further fractures. Unfortunately the
orthogeriatricians are all on annual leave and the consultant has asked you to arrange suitable management.
Which is the best option?

A. Alendronate

B. Alendronate, calcium and vitamin D supplementation

C. Strontium

D. Arrange a DEXA scan


E. Hormone replacement therapy

Next question

A bisphosphonate, calcium and vitamin D supplementation should be given to all patients aged over 75 years
after having a fracture. A DEXA scan is only needed of the patient is aged below 75 years. Hormone
replacement therpay has been shown to reduce vertebral and non vertebral fractures, however the risks of
cardiovascular disease and breast malignancy make this a less favourable option.

Osteoporosis: secondary prevention

NICE guidelines were updated in 2008 on the secondary prevention of osteoporotic fractures in
postmenopausal women.

Key points include

• Treatment is indicated following osteoporotic fragility fractures in postmenopausal women who are
confirmed to have osteoporosis (a T-score of - 2.5 SD or below).
• In women aged 75 years or older, a DEXA scan may not be required 'if the responsible clinician
considers it to be clinically inappropriate or unfeasible'
• Vitamin D and calcium supplementation should be offered to all women unless the clinician is
confident they have adequate calcium intake and are vitamin D replete
• Alendronate is first-line
• Around 25% of patients cannot tolerate alendronate, usually due to upper gastrointestinal problems.
These patients should be offered risedronate or etidronate (see treatment criteria below)
• Strontium ranelate and raloxifene are recommended if patients cannot tolerate bisphosphonates (see
treatment criteria below)

Supplementary notes on treatment

Bisphosphonates

• Alendronate, risedronate and etidronate are all licensed for the prevention and treatment of post-
menopausal and glucocorticoid-induced osteoporosis
• All three have been shown to reduce the risk of both vertebral and non-vertebral fractures although
alendronate, risedronate may be superior to etidronate in preventing hip fractures
• Ibandronate is a once-monthly oral bisphosphonate

Vitamin D and calcium


• Poor evidence base to suggest reduced fracture rates in the general population at risk of osteoporotic
fractures - may reduce rates in frail, housebound patients

Raloxifene - selective oestrogen receptor modulator (SERM)

• Has been shown to prevent bone loss and to reduce the risk of vertebral fractures, but has not yet been
shown to reduce the risk of non-vertebral fractures
• Has been shown to increase bone density in the spine and proximal femur
• May worsen menopausal symptoms
• Increased risk of thromboembolic events
• May decrease risk of breast cancer

Strontium ranelate

• 'Dual action bone agent' - increases deposition of new bone by osteoblasts and reduces the resorption
of bone by osteoclasts
• Strong evidence base, may be second-line treatment in near future
• Increased risk of thromboembolic events

Previous Question 38 of 83 Next


Which of the following statements relating to avascular necrosis is false?

A. When associated with fracture may occur despite the radiological evidence of
fracture union.

B. Pain and stiffness will typically precede radiological evidence of the condition.

C. Drilling of affected bony fragments may be used to facilitate angiogenesis


where arthroplasty is not warranted.

D. The earliest detectable radiological evidence is a radiolucency of the affected


area coupled with subchondral collapse.

E. It is less likely when prompt anatomical alignment of fracture fragments is


achieved.
Next question
Avascular necrosis- radiological changes occur late.

Radiolucency and subchondral collapse are late changes. The earliest evidence on plain films is the affected
area appearing as being more radio-opaque due to hyperaemia and resorption of the neighboring area. It may
be diagnosed earlier using bone scans and MRI.

Previous 2 / 3 Question 39-41 of 83 Next


Theme: Diseases affecting the spine

A. Spondylolysis
B. Spina bifida occulta
C. Spondylolisthesis
D. Meningomyelocele
E. Meningocele
F. Scoliosis - non structural
G. Scoliosis
H. Ankylosing spondylitis
I. Scheuermann's disease

Please select the most likely underlying diagnosis for the condition described. Each condition may be used
once, more than once or not at all.

39. A 19 year old female is involved in an athletics event. She has just completed the high jump when she
suddenly develops severe back pain and weakness affecting both her legs. on examination she has a
prominent sacrum and her lower back is painful.

Spondylolisthesis

Theme from September 2012 Exam


Young athletic females are the group most frequently affected by spondylolythesis who have a
background of spondylolysis. Whilst the latter condition is a risk factor for spondylolythesis the former
condition is most likely in a young athletic female who presents with sudden pain.

40. A 15 year old boy is brought to the clinic by his mother who is concerned that he has a mark overlying
his lower spine. On examination the boy has a patch of hair overlying his lower lumbar spine and a
birth mark at the same location. Lower limb neurological examination is normal.

Spina bifida occulta

Spina bifida occulta is a common condition and may affect up to 10% of the population. The more
severe types of spina bifida have more characteristic skin changes. Occasionally the unwary surgeon is
persuaded to operate on these "cutaneous" changes and we would advocate performing an MRI scan
prior to any such surgical procedure in this region.

41. A 19 year old female presents to the clinic with progressive pain in her neck and back. The condition
has been progressively worsening over the past 6 months. She has not presented previously because she
was an inpatient with a disease flare of ulcerative colitis. On examination she has a stiff back with
limited spinal extension on bending forwards.

You answered Spondylolysis

The correct answer is Ankylosing spondylitis

Ankylosing spondylitis is associated with HLA B27, there is a strong association with ulcerative colitis
in such individuals. The clinical findings are usually of a kyphosis affecting the cervical and thoracic
spine. Considerable symptomatic benefit may be obtained using non steroidal anti inflammatory drugs.
These should be used carefully in patients with inflammatory bowel disease who may be taking
steroids.

Next question

Diseases affecting the vertebral column

Ankylosing • Chronic inflammatory disorder affecting the axial skeleton


spondylitis • Sacro-ilitis is a usually visible in plain films
• Up to 20% of those who are HLA B27 positive will develop the condition
• Affected articulations develop bony or fibrous changes
• Typical spinal features include loss of the lumbar lordosis and progressive
kyphosis of the cervico-thoracic spine

Scheuermann's • Epiphysitis of the vertebral joints is the main pathological process


disease • Predominantly affects adolescents
• Symptoms include back pain and stiffness
• X-ray changes include epiphyseal plate disturbance and anterior wedging
• Clinical features include progressive kyphosis (at least 3 vertebrae must be
involved)
• Minor cases may be managed with physiotherapy and analgesia, more severe
cases may require bracing or surgical stabilisation

Scoliosis • Consists of curvature of the spine in the coronal plane


• Divisible into structural and non structural, the latter being commonest in
adolescent females who develop minor postural changes only. Postural
scoliosis will typically disappear on manoeuvres such as bending forwards
• Structural scoliosis affects > 1 vertebral body and is divisible into idiopathic,
congential and neuromuscular in origin. It is not correctable by alterations in
posture
• Within structural scoliosis, idiopathic is the most common type
• Severe, or progressive structural disease is often managed surgically with
bilateral rod stabilisation of the spine

Spina bifida • Non fusion of the vertebral arches during embryonic development
• Three categories; myelomeningocele, spina bifida occulta and meningocele
• Myelomeningocele is the most severe type with associated neurological
defects that may persist in spite of anatomical closure of the defect
• Up to 10% of the population may have spina bifida occulta, in this condition
the skin and tissues (but not not bones) may develop over the distal cord. The
site may be identifiable by a birth mark or hair patch
• The incidence of the condition is reduced by use of folic acid supplements
during pregnancy

Spondylolysis • Congenital or acquired deficiency of the pars interarticularis of the neural arch
of a particular vertebral body, usually affects L4/ L5
• May be asymptomatic and affects up to 5% of the population
• Spondylolysis is the commonest cause of spondylolisthesis in children
• Asymptomatic cases do not require treatment

Spondylolisthesis • This occurs when one vertebra is displaced relative to its immediate inferior
vertebral body
• May occur as a result of stress fracture or spondylolysis
• Traumatic cases may show the classic "Scotty Dog" appearance on plain films
• Treatment depends upon the extent of deformity and associated neurological
symptoms, minor cases may be actively monitored. Individuals with radicular
symptoms or signs will usually require spinal decompression and stabilisation
Previous 1 / 3 Question 42-44 of 83 Next
Theme: Management of fractures

A. Discharge home with arm sling and fracture clinic appointment


B. Discharge home with futura splint and fracture clinic appointment
C. Admit for open reduction and fixation
D. Fasciotomy
E. Active observation for progression of neurovascular compromise
F. Reduction of fracture in casualty and application of plaster backslab, followed
by discharge home.

Please select the most appropriate immediate management for the fracture scenarios given. Each option may be
used once, more than once or not at all.

42. A 22 year old rugby player falls onto an outstretched hand and sustains a fracture of the distal radius.
The x-ray shows a dorsally angulated comminuted fracture.

You answered Reduction of fracture in casualty and application of plaster backslab, followed by
discharge home.

The correct answer is Admit for open reduction and fixation

Unlike an osteoporotic fracture in an elderly lady this is a high velocity injury and will require surgical
fixation.

43. A 10 year old boy undergoes a delayed open reduction and fixation of a significantly displaced
supracondylar fracture. On the ward he complains of significant forearm pain and paraesthesia of the
hand. Radial pulse is normal.

You answered Admit for open reduction and fixation

The correct answer is Fasciotomy

The delay is the significant factor here. These injuries often have neurovascular compromise and
inactivity now places him at risk of developing complications. In compartment syndrome the loss of
arterial pulsation occurs late.
44. A 28 year old man falls onto an outstretched hand. On examination there is tenderness of the anatomical
snuffbox. However, forearm and hand x-rays are normal.

Discharge home with futura splint and fracture clinic appointment

This could well be a scaphoid fracture and should be temporarily immobilised pending further review.
A futura splint will immobilise better than an arm sling for this problem.

Next question

Fracture management

• Bony injury resulting in a fracture may arise from trauma (excessive forces applied to bone), stress
related (repetitive low velocity injury) or pathological (abnormal bone which fractures during normal
use of following minimal trauma)
• Diagnosis involves not just evaluating the fracture ; such as site and type of injury but also other
associated injuries and distal neurovascular deficits. This may entail not just clinical examination but
radiographs of proximal and distal joints.
• When assessing x-rays it is important to assess for changes in length of the bone, the angulation of the
distal bone, rotational effects, presence of material such as glass.

Fracture types

Fracture type Description

Oblique fracture Fracture lies obliquely to long axis of bone

Comminuted fracture >2 fragments

Segmental fracture More than one fracture along a bone

Transverse fracture Perpendicular to long axis of bone

Spiral fracture Severe oblique fracture with rotation along long axis of bone

Open Vs Closed
It is also important to distinguish open from closed injuries. The most common classification system for open
fractures is the Gustilo and Anderson classification system (given below):
Grade Injury

1 Low energy wound <1cm

2 Greater than 1cm wound with moderate soft tissue damage

3 High energy wound > 1cm with extensive soft tissue damage

3 A (sub group of 3) Adequate soft tissue coverage

3 B (sub group of 3) Inadequate soft tissue coverage

3 C (sub group of 3) Associated arterial injury

Key points in management of fractures

• Immobilise the fracture including the proximal and distal joints


• Carefully monitor and document neurovascular status, particularly following reduction and
immobilisation
• Manage infection including tetanus prophylaxis
• IV broad spectrum antibiotics for open injuries
• As a general principle all open fractures should be thoroughly debrided ( and internal fixation devices
avoided or used with extreme caution)
• Open fractures constitute an emergency and should be debrided and lavaged within 6 hours of injury

Question 45 of 83 Next
A 4 year old boy falls and sustains a fracture to the growth plate of his right wrist. Which of the following
systems is used to classify the injury?

A. Salter - Harris system

B. Weber system

C. Gustilo - Anderson system

D. Garden system

E. None of the above

Next question
The Salter - Harris system is most commonly used. The radiological signs in Type 1 and 5 injuries may be
identical. Which is unfortunate as type 5 injuries do not do well (and may be missed!)

Previous 3 / 3 Question 46-48 of 83 Next


Theme: Bone disease

A. Osteogenesis imperfecta
B. Osteoporosis
C. Rickets
D. Pagets disease
E. Chondrosarcoma
F. Metastatic breast cancer

Please select the most likely diagnosis for the scenario given. Each option may be used once, more than once
or not at all.

46. A 66 year old lady presents with pain in her right hip. It has been increasing over the previous three
weeks and waking her from sleep. On examination she is tender on internal rotation. Blood tests reveal
a mildly elevated serum calcium and alkaline phosphatase levels.

Metastatic breast cancer

Increasing pain at rest, together with increased serum calcium and alkaline phosphatase are most likely
to represent metastatic tumour to bone. Chondrosarcomas do occur in the pelvis but are not associated
with increased serum calcium and typically have a longer history.

47. A 73 year old man presents with pain in the right leg. It is most uncomfortable on walking. On
examination he has a deformity of his right femur, which on x-ray is thickened and sclerotic. His serum
alkaline phosphatase is elevated, but calcium is within normal limits.

Pagets disease

This is a typical scenario for Pagets disease.

48. A 73 year old lady presents with pain in her left hip. She was walking around the house when she
tripped over a rug and fell over. Apart from temporal arteritis which is well controlled with
prednisolone she is otherwise well. On examination he leg is shorted and externally rotated.Her serum
alkaline phosphatase and calcium are normal.

Osteoporosis

The combination of age, female gender and steroids coupled with hip pain on minor trauma are strongly
suggestive of osteoporosis.

Next question

Bone disease

Disease Features Treatment

Pagets • Focal bone resorption followed by Bisphosphonates


excessive and chaotic bone deposition
• Affects (in order): spine, skull, pelvis and
femur
• Serum alkaline phosphatase raised (other
parameters normal)
• Abnormal thickened, sclerotic bone on x-
rays
• Risk of cardiac failure with >15% bony
involvement
• Small risk of sarcomatous change

Osteoporosis • Excessive bone resorption resulting in Bisphosphonates, calcium and


demineralised bone vitamin D
• Commoner in old age
• Increased risk of pathological fracture,
otherwise asymptomatic
• Alkaline phosphatase normal, calcium
normal

Secondary bone • Bone destruction and tumour infiltration Radiotherapy, prophylactic


tumours • Mirel scoring used to predict risk of fixation and analgesia
fracture
• Appearances depend on primary
(e.g.sclerotic - prostate, lytic - breast)
• Elevated serum calcium and alkaline
phosphatase may be seen

Previous 2 / 3 Question 49-51 of 83 Next


Theme: Shoulder pain

A. Impingement syndrome
B. Rotator cuff tear
C. Adhesive capsulitis
D. Calcific tendonitis
E. Biceps tendon rupture
F. Parsonage - Turner syndrome
G. Labral tear

Please select the most likely cause for shoulder pain from the list. Each option may be used once, more than
once or not at all.

49. A 63 year old lady undergoes an axillary clearance for breast cancer. She makes steady progress.
However, 8 weeks post operatively she still suffers from severe shoulder pain. On examination she has
reduced active movements in all planes and loss of passive external rotation.

Adhesive capsulitis

Frozen shoulder passes through an initial painful stage followed by a period of joint stiffness. With
physiotherapy the problem will usually resolve although it may take up to 2 years to do so.

50. A 78 year old man complains of a long history of shoulder pain and more recently weakness. On
examination active attempts at abduction are impaired. Passive movements are normal.

Rotator cuff tear

Rotator cuff tears are common in elderly people and may occur following minor trauma or as a result of
long standing impingement. Tears greater than 2cm should generally be repaired surgically.

51. A 28 year old man complains of pain and weakness in the shoulder. He has recently been unwell with
glandular fever from which he is fully recovered. On examination there is some evidence of muscle
wasting and a degree of winging of the scapula. Power during active movements is impaired.
You answered Impingement syndrome

The correct answer is Parsonage - Turner syndrome

This is a peripheral neuropathy that may complicate viral illnesses and usually resolves spontaneously.

Next question

Deep seated pain in the proximal forearm especially during the night and at rest may be due to tumour,
especially metastatic lesions.

Shoulder disorders

Proximal humerus fractures

• Very common. Usually through the surgical neck. Number of classification systems though for
practical purposes describing the number of fracture fragments is probably easier. Some key points:
• It is rare to have fractures through the anatomical neck.
• Anatomical neck fractures which are displaced by >1cm carry a risk of avascular necrosis to the
humeral head.
• In children the commonest injury pattern is a greenstick fracture through the surgical neck.
• Impacted fractures of the surgical neck are usually managed with a collar and cuff for 3 weeks
followed by physiotherapy.
• More significant displaced fractures may require open reduction and fixation or use of an
intramedullary device.

Types of shoulder dislocation

• Glenohumeral dislocation (commonest): anterior shoulder dislocation most common


• Acromioclavicular dislocation (12%): clavicle loses all attachment with the scapula
• Sternoclavicular dislocation (uncommon)

Types of glenohumeral dislocation:

Anterior shoulder • External rotation and abduction


dislocation • 35-40% recurrent (it is the commonest disorder)
• Associated with greater tuberosity fracture, Bankart lesion, Hill-Sachs
defect

Inferior shoulder Luxatio erecta


dislocation

Posterior shoulder • Proportion misdiagnosed.


dislocation • Rim's sign, light bulb sign.
• Associated with Trough sign

Superior shoulder Rare and usually follow major trauma.


dislocation

Treatment
Prompt reduction is the mainstay of treatment and is usually performed in the emergency department.
Neurovascular status must be checked pre and post reduction and x-rays should be performed again post
reduction to ensure no fracture has occurred. In recurrent anterior dislocation there is usually a Bankart lesion
and this may be repaired surgically. Recurrent posterior dislocations may be repaired in a similar manner to
anterior lesions but using a posterior (or arthroscopic) approach.

Previous 0 / 3 Question 52-54 of 83 Next


Theme: Knee injuries

A. Anterior cruciate ligament rupture


B. Posterior cruciate ligament rupture
C. Medial collateral ligament tear
D. Lateral collateral ligament tear
E. Torn meniscus
F. Chondromalacia patellae
G. Dislocated patella
H. Fractured patella
I. Tibial plateau fracture

What is the most likely injury for scenario given? Each option may be used once, more than once or not at all.
52. A 38 year old man is playing football when he slips over during a tackle. His knee is painful
immediately following the fall. Several hours later he notices that the knee has become swollen.
Following a course of non steroidal anti inflammatory drugs and rest the situation improves. However,
complains of recurrent pain. On assessment in clinic you notice that it is impossible to fully extend the
knee, although the patient is able to do so when asked.

You answered Tibial plateau fracture

The correct answer is Torn meniscus

Theme from September 2012 Exam


Twisting sporting injuries followed by delayed onset of knee swelling and locking are strongly
suggestive of a menisceal tear. Arthroscopic menisectomy is the usual treatment.

53. A 34 year old woman is a passenger in a car during an accident. Her knee hits the dashboard. On
examination the tibia looks posterior compared to the non injured knee.

You answered Torn meniscus

The correct answer is Posterior cruciate ligament rupture

In ruptured posterior cruciate ligament the tibia lies back on the femur and can be drawn forward during
a paradoxical draw test.

54. A 28 year old professional footballer is admitted to the emergency department. During a tackle he is
twisted with his knee flexed. He hears a loud crack and his knee rapidly becomes swollen.

You answered Fractured patella

The correct answer is Anterior cruciate ligament rupture

This is common in footballers as the football boot studs stick to the ground and high twisting force is
applied to a flexed knee. Rapid joint swelling also supports the diagnosis.

Next question

Knee injury

Types of injury
Ruptured anterior • Sport injury
cruciate ligament • Mechanism: high twisting force applied to a bent knee
• Typically presents with: loud crack, pain and RAPID swelling knee
(haemoarthrosis)
• Poor healing
• Management: intense physiotherapy or surgery

Ruptured posterior • Mechanism: hyperextension injuries


cruciate ligament • Tibia lies back on the femur
• Paradoxical anterior draw test

Rupture of medial • Mechanism: leg forced into valgus via force outside the leg
collateral ligament • Knee unstable when put into valgus position

Menisceal tear • Rotational sporting injuries


• Delayed knee swelling
• Joint locking (Patient may develop skills to "unlock" the knee
• Recurrent episodes of pain and effusions are common, often following
minor trauma

Chondromalacia • Teenage girls, following an injury to knee e.g. Dislocation patella


patellae • Typical history of pain on going downstairs or at rest
• Tenderness, quadriceps wasting

Dislocation of the • Most commonly occurs as a traumatic primary event, either through
patella direct trauma or through severe contraction of quadriceps with knee
streched in valgus and external rotation
• Genu valgum, tibial torsion and high riding patella are risk factors
• Skyline x-ray views of patella are required, although displaced patella
may be clinically obvious
• An osteochondral fracture is present in 5%
• The condition has a 20% recurrence rate

Fractured patella • 2 types:

i. Direct blow to patella causing undisplaced fragments


ii. Avulsion fracture
Tibial plateau fracture • Occur in the elderly (or following significant trauma in young)
• Mechanism: knee forced into valgus or varus, but the knee fractures
before the ligaments rupture
• Varus injury affects medial plateau and if valgus injury, lateral plateau
depressed fracture occurs
• Classified using the Schatzker system (see below)

Schatzker Classification system for tibial plateau fractures


Type Features
1- vertical split of lateral condyle Fracture through dense bone, usually in the young. It may be virtually
undisplaced, or the condylar fragment may be pushed inferiorly and tilted
2- a vertical split of the lateral The wedge fragement (which may be of variable size), is displaced
condyle combined with an laterally; the joint is widened. Untreated, a valgus deformity may develop
adjacent loadbearing part of the
condyle
3- depression of the articular The split does not extend to the edge of the plateau. Depressed fragments
surface with intact condylar rim may be firmly embedded in subchondral bone, the joint is stable
4- fragment of the medial tibial Two injuries are seen in this category; (1) a depressed fracture of
condyle osteoporotic bone in the elderly. (2) a high energy fracture resulting in a
condylar split that runs from the intercondylar eminence to the medial
cortex. Associated ligamentous injury may be severe
5-fracture of both condyles Both condyles fractured but the column of the metaphysis remains in
continuity with the tibial shaft
6-combined condylar and High energy fracture with marked comminution
subcondylar fractures

Previous Question 55 of 83 Next


A 10 year old boy is referred to the orthopaedic clinic with symptoms of right knee pain. He has suffered pain
for the past 3 months and the pain typically lasts for several hours. On examination he walks with an antalgic
gait and has apparent right leg shortening. The right knee is normal but the right hip reveals pain on internal
and external rotation. Imaging shows flattening of the femoral head. Which of the following is the most likely
underlying diagnosis?

A. Osteogenesis imperfecta

B. Child abuse

C. Osteosarcoma
D. Osteopetrosis

E. Perthes disease

Next question

This is a typical description of Perthes disease. Management involves keeping the femoral head in the
acetabulum by braces, casts or surgery.

Previous Question 56 of 83 Next


Which statement relating to talipes equinovarus is untrue?

A. It has an annual incidence of around 1 in 1000 in the UK.

B. The muscles involved in the disorder are intrinsically abnormal.

C. The cuboid is classically displaced medially.

D. All cases should be treated with an Ilizarov frame initially unless there is
minor deformity.

E. The talocalcaneal angle is typically less than 20 degrees in club foot.

Next question

In most cases of Club Foot conservative measures should be tried first. The Ponsetti method is a popular
approach. Severe cases may benefit from Ilizarov frame re-aligment.

Talipes Equinovarus

Congenital talipes equinovarus.


Features:

• Equinus of the hindfoot.


• Adduction and varus of the midfoot.
• High arch.

Most cases in developing countries. Incidence in UK is 1 per 1000 live births. It is more common in males and
is bilateral in 50% cases. There is a strong familial link(1). It may also be associated with other developmental
disorders such as Down's syndrome.
Key anatomical deformities (2):

• Adducted and inverted calcaneus


• Wedge shaped distal calcaneal articular surface
• Severe Tibio-talar plantar flexion.
• Medial Talar neck inclination
• Displacement of the navicular bone (medially)
• Wedge shaped head of talus
• Displacement of the cuboid (medially)

Management
Conservative first, the Ponseti method is best described and gives comparable results to surgery. It consists of
serial casting to mold the foot into correct shape. Following casting around 90% will require a Achilles
tenotomy. This is then followed by a phase of walking braces to maintain the correction.

Surgical correction is reserved for those cases that fail to respond to conservative measures. The procedures
involve multiple tenotomies and lengthening procedures. In patients who fail to respond surgically an Ilizarov
frame reconstruction may be attempted and gives good results.

Previous Question 57 of 83 Next


Which of the following is least likely to impair bone fracture healing?

A. Radiotherapy

B. Osteoporosis

C. Administration of non steroidal anti inflammatory drugs

D. Preservation of periosteum

E. Presence of osteomyelitic sequestra

Next question

Periosteal preservation helps fractures to heal.

Fracture healing
Bone fracture
- Bleeding vessels in the bone and periosteum
- Clot and haematoma formation
- The clot organises over a week (improved structure and collagen)
- The periosteum contains osteoblasts which produce new bone
- Mesenchymal cells produce cartilage (fibrocartilage and hyaline cartilage) in the soft tissue around the
fracture
- Connective tissue + hyaline cartilage = callus
- As the new bone approaches the new cartilage, endochondral ossification occurs to bridge the gap
- Trabecular bone forms
- Trabecular bone is resorbed by osteoclasts and replaced with compact bone

Factors Affecting Fracture Healing

• Age
• Malnutrition
• Bone disorders: osteoporosis
• Systemic disorders: diabetes, Marfan's syndrome and Ehlers-Danlos syndrome cause abnormal
musculoskeletal healing.
• Drugs: steroids, non steroidal anti inflammatory agents.
• Type of bone: Cancellous (spongy) bone fractures are usually more stable, involve greater surface
areas, and have a better blood supply than cortical (compact) bone fractures.
• Degree of Trauma: The more extensive the injury to bone and surrounding soft tissue, the poorer the
outcome.
• Vascular Injury: Especially the femoral head, talus, and scaphoid bones.
• Degree of Immobilization
• Intra-articular Fractures: These fractures communicate with synovial fluid, which contains
collagenases that retard bone healing.
• Separation of Bone Ends: Normal apposition of fracture fragments is needed for union to occur.
Inadequate reduction, excessive traction, or interposition of soft tissue will prevent healing.
• Infection

Previous 1 / 3 Question 58-60 of 83 Next


Theme: Disorders of the hip

A. Perthes disease
B. Developmental dysplasia of the hip
C. Osteoarthritis
D. Slipped upper femoral epiphysis
E. Septic arthritis
F. Rheumatoid arthritis
G. Intra capsular fracture of the femoral neck
H. Extra capsular fracture of the femoral neck

Please select the most likely diagnosis for the scenario given. Each option may be used once, more than once
or not at all.

58. An obese 14 year old boy presents with difficulty running and mild knee and hip pain. There is no
antecedent history of trauma. On examination internal rotation is restricted but the knee is normal with
full range of passive movement possible and no evidence of effusions. Both the C-reactive protein and
white cell count are normal.

You answered Osteoarthritis

The correct answer is Slipped upper femoral epiphysis

Slipped upper femoral epiphysis is the commonest adolescent hip disorder. It occurs most commonly in
obese males. It may often present as knee pain which is usually referred from the ipsilateral hip. The
knee itself is normal. The hip often limits internal rotation. The diagnosis is easily missed. X-rays will
show displacement of the femoral epiphysis and the degree of its displacement may be calculated using
the Southwick angle. Treatment is directed at preventing further slippage which may result in avascular
necrosis of the femoral head.

59. A 6 year old boy presents with pain in the hip it is present on activity and has been worsening over the
past few weeks. There is no history of trauma. He was born by normal vaginal delivery at 38 weeks
gestation On examination he has an antalgic gait and limitation of active and passive movement of the
hip joint in all directions. C-reactive protein is mildly elevated at 10 but the white cell count is normal.

You answered Slipped upper femoral epiphysis

The correct answer is Perthes disease

This is a typical presentation for Perthes disease. X-ray may show flattening of the femoral head or
fragmentation in more advanced cases.

60. A 30 year old man presents with severe pain in the left hip it has been present on and off for many
years. He was born at 39 weeks gestation by emergency caesarean section after a long obstructed breech
delivery. He was slow to walk and as a child was noted to have an antalgic gait. He was a frequent
attender at the primary care centre and the pains dismissed as growing pains. X-rays show almost
complete destruction of the femoral head and a narrow acetabulum.

Developmental dysplasia of the hip

Developmental dysplasia of the hip. Usually diagnosed by Barlow and Ortolani tests in early childhood.
Most Breech deliveries are also routinely subjected to USS of the hip joint. At this young age an
arthrodesis may be preferable to hip replacement.

Next question

Early plain x-ray changes in Perthes Disease:


Widening of the joint space.
Sub chondral linear lucency.

Previous 2 / 3 Question 61-63 of 83 Next


Theme: Paediatric fractures

A. Non accidental injury


B. Accidental fracture
C. Rickets
D. Metabolic bone disease of prematurity
E. Hypophosphataemic rickets
F. Osteopetrosis
G. Osteogenesis imperfecta
H. Hypoparathyroidism
I. Osteoporosis

Please select the most likely explanation for each of the following injury scenarios. Each option may be used
once, more than once or not at all.

61. A toddler aged 3 years presents to the Emergency Department with swelling of his leg and is found to
have a spiral fracture of the tibia. His mother reports that he had tripped and fallen the previous day but
she had not noticed any sign of injury at the time. She is a single parent with little family support. The
child is not on the child protection register.

Non accidental injury

Delayed presentation is unusual and should raise concern. In addition spiral fractures are usually the
result of rotational injury which is not compatible with the mechanism proposed by the parent.

62. A 5 month baby boy presents with swelling of his right arm and is found to have a spiral fracture of the
humerus. He had been in the care of her mother's boyfriend who reported that he had nearly dropped
her that day when reaching for his bottle and had inadvertently pulled on his arm to save him. He was
immediately taken to the Emergency Department.

Accidental fracture

The mechanism fits with the fracture pattern and the presentation is not delayed.

63. An infant is admitted with symptoms and signs of respiratory infection and is found to have several
posterior rib fractures on chest radiograph. He was born prematurely at 37 weeks' gestation and was
observed overnight on the special care baby unit for tachypnoea which settled by the following day. On
assessment it is also apparent that his head circumference has increased at an excessive rate and has
crossed 3 centiles since birth.

You answered Metabolic bone disease of prematurity

The correct answer is Non accidental injury

Posterior rib fractures are extremely unusual in neonates. The change in head size may be accounted for
by hydrocephalus which may occur as a sequelae from head injury.

Previous Question 64 of 83 Next


In paediatric orthopaedic surgery, which of the following does not fulfill the Kocher criteria for septic arthritis?

A. ESR > 40mm/h

B. Positive blood culture

C. Fever
D. White cell count > 12, 000

E. Non weight bearing on the affected side

Next question

Kocher criteria
1. Non weight bearing on affected side
2. ESR > 40 mm/hr
3. Fever
4. WBC count of >12,000 mm3
- When 4/4 criteria are met, there is a 99% chance that the child has septic arthritis

The Kocher criteria do not consider blood culture results.

Septic arthritis- Paediatric

Septic arthritis

• Staph aureus commonest organism


• Urgent washout and antibiotics otherwise high risk of joint destruction

Diagnosis

• Plain x-rays
• Consider aspiration
• Utilise the Kocher criteria (see below)

Kocher criteria:
1. Non weight bearing on affected side
2. ESR > 40 mm/hr
3. Fever
4. WBC count of >12,000 mm3
- when 4/4 criteria are met, there is a 99% chance that the child has septic arthritis

Treatment
Surgical drainage of the affected joint is required, this should be done as soon as possible since permanent
damage to the joint may occur. In some cases repeated procedures are necessary. Appropriate intravenous
antibiotics should be administered.
Previous 2 / 3 Question 65-67 of 83 Next
Theme: Ankle fractures

A. Surgical fixation
B. Below knee amputation
C. Aircast boot
D. Application of full leg plaster cast to include midfoot
E. Application of below knee plaster cast to include the midfoot
F. Application of external fixation device
G. Application of compression bandage and physiotherapy.

Please select the most appropriate management for the injury type described. Each option may be used once,
more than once or not at all.

65. A 24 year old man falls sustaining an inversion injury to his ankle. On examination he is tender over the
lateral malleolus only. On x-ray there is a fibular fracture that is distal to the syndesmosis.

Application of below knee plaster cast to include the midfoot

Theme from 2008 Exam


These distal injuries are generally managed conservatively. Conservative management will involve a
below knee cast, this will need to extend to the midfoot. It can be substituted for an aircast boot once
radiological union is achieved.

66. An 86 year old lady stumbles and falls whilst opening her front door. On examination her ankle is
swollen with both medial and lateral tenderness. X rays demonstrate a fibular fracture at the level of the
syndesmosis.

Application of below knee plaster cast to include the midfoot

Although, this is a potentially unstable injury operative fixation in this age group generally gives poor
results owing to poor quality bone. A below knee cast should be applied in the first instance. If this fails
to provide adequate control it can be extended above the knee.

67. A 25 year old man suffers an injury whilst playing rugby involving a violent twist to his left lower leg.
On examination both malleoli are tender and the ankle joint is very swollen. On x-ray there is a spiral
fracture of the fibula and widening of the ankle mortise.
You answered Application of external fixation device

The correct answer is Surgical fixation

This is a variant of the Weber C fracture in which disruption of the tibio-fibular syndesmosis occurs
leading to joint disruption. Surgical repair is warranted.

Next question

Ankle injuries

Ankle fractures are a common cause of admission to casualty. Clinical examination is facilitated by
the Ottawa ankle rules to try and minimise the unnecessary use of x-rays.

• These state that x-rays are only necessary if there is pain in the malleolar zone and:

1. Inability to weight bear for 4 steps


2. Tenderness over the distal tibia
3. Bone tenderness over the distal fibula

A number of classification systems exist for describing ankle fractures, these include the Potts, Weber and AO
systems. For simplicity the Weber system is outlined here.

Weber classification
Related to the level of the fibular fracture.

• Type A is below the syndesmosis


• Type B fractures start at the level of the tibial plafond and may extend proximally to involve the
syndesmosis
• Type C is above the syndesmosis which may itself be damaged

A subtype known as a Maisonneuve fracture may occur with spiral fibular fracture that leads to disruption of
the syndesmosis with widening of the ankle joint, surgery is required.

Management
Depends upon stability of ankle joint and patient co-morbidities.
All ankle fractures should be promptly reduced to remove pressure on the overlying skin and subsequent
necrosis.
Young patients, with unstable, high velocity or proximal injuries will usually require surgical repair. Often
using a compression plate.
Elderly patients, even with potentially unstable injuries usually fare better with attempts at conservative
management as their thin bone does not hold metalwork well.

Previous 2 / 3 Question 68-70 of 83 Next


Theme: Management of hip fractures

A. Hemiarthroplasty cemented prosthesis


B. Hemiarthroplasty non cemented prosthesis
C. Percutaneous pinning
D. Conservative management
E. Dynamic hip screw
F. Intramedullary device
G. Hip arthrodesis
H. Total hip replacement

For each fracture scenario please select the most appropriate management option from the list. Each option
may be used once, more than once or not at all.

68. A 72 year old retired teacher is admitted to A&E with a fall and hip pain. He is normally fit and well.
He lives with his son in a detached, 2 storey house. A hip x-ray confirms an extracapsular fracture.

Dynamic hip screw

Extracapsular fractures should be treated surgically. Since the blood supply to the femoral head is not
compromised joint replacement is not usually warranted.

69. A 72 year old retired teacher is admitted to A&E with a fall and hip pain. He is normally fit and well.
He lives with his son in a detached, 2 storey house. A hip x-ray confirms an subtrochanteric fracture.

Intramedullary device

Intramedullary device is normally recommended for reverse oblique, transverse or subtrochanteric


fractures.
70. An 86 year old retired pharmacist is admitted to A&E following a fall. She complains of right hip pain.
She is known to have hypertension and is currently on bendrofluazide. She lives alone and does not
mobilise. Her right leg is shortened and externally rotated. A hip x-ray confirms a displaced
intracapsular fracture.

You answered Total hip replacement

The correct answer is Hemiarthroplasty non cemented prosthesis

This patient warrants a hemiarthroplasty due to reduced mobility and older age. The anterolateral
approach is recommended in the SIGN guidelines. In this case most surgeons would not use a cemented
prosthesis.

Previous 0 / 3 Question 71-73 of 83 Next


Theme: Paediatric orthopaedics

A. USS hip
B. Hip x-ray
C. Anteroposterior pelvic x-ray
D. CT scan
E. MRI scan
F. Technetium bone scan
G. USS knee
H. X-ray knee
I. Discharge and reassure

For each of the following scenarios which is the most appropriate investigation? Each option may be used
once, more than once or not at all.

71. An obese 12 year old boy presents with knee pain. On examination he has pain on internal rotation of
the hip. His knee is clinically normal.

You answered USS hip

The correct answer is Hip x-ray

The main differential diagnosis in a boy over 10 years old is of slipped upper femoral epiphysis. Knee
pain is a common presenting feature. An anteroposterior pelvic x-ray may miss a minor slip, therefore
request a hip film.

72. A baby is delivered in the breech position. Barlows and Ortolani tests are normal

You answered Discharge and reassure

The correct answer is USS hip

This child is at risk of developmental dysplasia of the hip (up to 20% will have DDH), so should have
the hip joints scanned to exclude this.

73. A 5 year old boy presents with a painful limp. The symptoms have been present for 8 weeks. Two hip
x-rays have been performed and appear normal.

You answered X-ray knee

The correct answer is Technetium bone scan

Perthes disease should be suspected in boys over 4 years old presenting with a limp. Early disease can
be missed on x-ray, therefore a bone scan should be performed. MRI is less sensitive than the bone
scan.

Next question
Previous Question 74 of 83 Next
A 5 year old boy is playing in a tree when he falls and lands on his right forearm. He is brought to the
emergency department by his parents. On examination he has bony tenderness and bruising. An X-ray is taken
and shows unilateral cortical disruption and development of periosteal haematoma. What is the most likely
diagnosis?

A. Buckle fracture

B. Greenstick fracture

C. Toddlers fracture

D. Complete fracture

E. None of the above


Greenstick fractures are common childhood injuries. Unilateral cortical disruption is the main radiological
feature, since involvement of both cortices makes the injury a complete fracture. Buckle fractures will show
periosteal haematoma formation only.

Previous 0 / 3 Question 75-77 of 83 Next


Theme: Eponymous fractures

A. Smith's
B. Bennett's
C. Monteggia's
D. Colle's
E. Galeazzi
F. Pott's
G. Barton's

Which is the most likely eponymous fracture for the scenario given. Each option may be used once, more than
once or not at all.

75. A 14 year old boy jumps off a 10 foot wall and lands on both feet. An x-ray shows a bimalleolar
fracture of the right ankle.

You answered Barton's

The correct answer is Pott's

76. A 22 year old drunk man is involved in a fight. He hurts his thumb when he punches his opponent.

You answered Galeazzi

The correct answer is Bennett's

77. A 63 year nurse falls on an extended and pronated wrist. An x-ray shows a distal radial fracture
with radiocarpal dislocation.

You answered Colle's


The correct answer is Barton's

Previou
1/3 Question 78-80 of 83 Next
s
Theme: Fracture management

A. Application of external fixator


B. Open reduction and internal fixation
C. Fasciotomy
D. Skeletal traction

For the following upper limb injuries please select the most appropriate initial management. Each option may
be used once, more than once or not at all.

78. A 32 year old man falls from a ladder and sustains a fracture of his proximal radius. On examination he
has severe pain in his forearm and diminished distal sensation. There is a single puncture wound present
at the fracture site.

Fasciotomy

Theme from April 2012 Exam


Pain and neurological symptoms in a tight fascial compartment coupled with a high velocity injury
carry a high risk of compartment syndrome and prompt fasciotomy should be performed.

79. A 32 year old man falls a sustains a fracture of his distal humerus. The fracture segment is markedly
angulated and unstable. There is a puncture site overlying the fracture site.

You answered Open reduction and internal fixation

The correct answer is Application of external fixator

Wide exposure to plate the humerus is generally inadvisable owing to its many important anatomical
relations. Both intramedullary nailing and external fixation are reasonable treatments. However, in the
presence of an open fracture application of an external fixator and appropriate tissue debridement would
be most appropriate.

80. A 24 year old man sustains a distal radius fracture during a game of rugby. Imaging shows a
comminuted fracture with involvement of the articular surface.

You answered Application of external fixator

The correct answer is Open reduction and internal fixation

Meticulous anatomical alignment of the fracture segments is crucial to avoid the development of
osteoarthritis and risk of malunion.

Next question

Fracture management

• Bony injury resulting in a fracture may arise from trauma (excessive forces applied to bone), stress
related (repetitive low velocity injury) or pathological (abnormal bone which fractures during normal
use of following minimal trauma)
• Diagnosis involves not just evaluating the fracture ; such as site and type of injury but also other
associated injuries and distal neurovascular deficits. This may entail not just clinical examination but
radiographs of proximal and distal joints.
• When assessing x-rays it is important to assess for changes in length of the bone, the angulation of the
distal bone, rotational effects, presence of material such as glass.

Fracture types

Fracture type Description

Oblique fracture Fracture lies obliquely to long axis of bone

Comminuted fracture >2 fragments

Segmental fracture More than one fracture along a bone

Transverse fracture Perpendicular to long axis of bone

Spiral fracture Severe oblique fracture with rotation along long axis of bone

Open Vs Closed
It is also important to distinguish open from closed injuries. The most common classification system for open
fractures is the Gustilo and Anderson classification system (given below):
Grade Injury

1 Low energy wound <1cm

2 Greater than 1cm wound with moderate soft tissue damage

3 High energy wound > 1cm with extensive soft tissue damage

3 A (sub group of 3) Adequate soft tissue coverage

3 B (sub group of 3) Inadequate soft tissue coverage

3 C (sub group of 3) Associated arterial injury

Key points in management of fractures

• Immobilise the fracture including the proximal and distal joints


• Carefully monitor and document neurovascular status, particularly following reduction and
immobilisation
• Manage infection including tetanus prophylaxis
• IV broad spectrum antibiotics for open injuries
• As a general principle all open fractures should be thoroughly debrided ( and internal fixation devices
avoided or used with extreme caution)
• Open fractures constitute an emergency and should be debrided and lavaged within 6 hours of injury

• Previous 1/3 Question 81-83 of 83


Theme: Fracture management

A. Copious lavage and generous surgical debridement, followed by external


fixation
B. Intramedullary nail
C. Open reduction and internal fixation
D. Immobilisation in plaster cast
E. External fixation using a frame device
F. Amputation
G. Application of external fixation device
H. Primary closure of wound and application of plaster cast
Please select the most appropriate management for the fractures described. Each option may be used once,
more than once or not at all.

81 A 55 year old motorcyclist is involved in a road traffic accident and sustained a Gustilo and Anderson
. IIIc type fracture to the distal tibia. He was trapped in the wreckage for 7 hours during which time he
bled profusely from the fracture site. He has an established distal neurovascular deficit.

You answered Copious lavage and generous surgical debridement, followed by external fixation

The correct answer is Amputation

This man is unstable, and at 7 hours after extraction, the limb is not viable. The safest option is primary
amputation.

82 A 25 year old ski instructor who falls off a ski lift and sustains a spiral fracture of the mid shaft of the
. tibia. Attempts to achieve satisfactory position in plaster have failed. Overlying tissues are healthy.

Intramedullary nail

This would be a good case for intramedullary nailing. Open reduction and external fixation would strip
off otherwise healthy tissues and hence is unsuitable. In some units the injury may be managed with an
Ilizarov frame device but the majority would treat with IM nailing.

83 A 35 year old mechanic is hit by a fork lift truck. He sustains a Gustilo and Anderson type IIIA
. fracture of the shaft of the left femur.

You answered Primary closure of wound and application of plaster cast

The correct answer is Copious lavage and generous surgical debridement, followed by external
fixation

At the tissues are in better shape than in the first case and as there is no associated vascular injury the
patient may be suitable for debridement of the area and external fixation. If debridement leaves a tissue
defect then plastic surgical repair will be needed at a later stage.

Delayed treatment of open fractures with significant vascular injury may be best treated by primary
amputation.

You might also like